Stay ahead of learning milestones! Enroll in a class over the summer!

G
Topic
First Poster
Last Poster
Three lines meet at one point
TUAN2k8   0
10 minutes ago
Source: Own
Let $ABC$ be an acute triangle incribed in a circle $\omega$.Let $M$ be the midpoint of $BC$.Let $AD,BE$ and $CF$ be altitudes from $A,B$ and $C$ of triangle $ABC$, respectively, and let them intersect at $H$.Let $K$ be the intersection point of tangents to the circle $\omega$ at points $B,C$.Prove that $MH,KD$ and $EF$ are concurrent.
0 replies
TUAN2k8
10 minutes ago
0 replies
Geometry with altitudes and the nine point centre
Adywastaken   1
N 24 minutes ago by Adywastaken
Source: KoMaL B5333
The foot of the altitude from vertex $A$ of acute triangle $ABC$ is $T_A$. The ray drawn from $A$ through the circumcenter $O$ intersects $BC$ at $R_A$. Let the midpoint of $AR_A$ be $F_A$. Define $T_B$, $R_B$, $F_B$, $T_C$, $R_C$, $F_C$ similarly. Prove that $T_AF_A$, $T_BF_B$, $T_CF_C$ are concurrent.
1 reply
Adywastaken
24 minutes ago
Adywastaken
24 minutes ago
This question just asks if you can factorise 12 factorial or not
Sadigly   5
N 38 minutes ago by Just1
Source: Azerbaijan Junior MO 2025 P1
A teacher creates a fraction using numbers from $1$ to $12$ (including $12$). He writes some of the numbers on the numerator, and writes $\times$ (multiplication) between each number. Then he writes the rest of the numbers in the denominator and also writes $\times$ between each number. There is at least one number both in numerator and denominator. The teacher ensures that the fraction is equal to the smallest possible integer possible.

What is this positive integer, which is also the value of the fraction?
5 replies
Sadigly
May 9, 2025
Just1
38 minutes ago
Inequality for beginners.
mudok   3
N 42 minutes ago by sqing
Source: own
$a,b,c>0, \ \ \ \sqrt{a}+\sqrt{b}+\sqrt{c}=3$. Prove that \[\sqrt{a^2+1}+\sqrt{b^2+1}+\sqrt{c^2+1}\le \sqrt{2}(a+b+c)\]
I meant easy inequality...
3 replies
mudok
Aug 6, 2012
sqing
42 minutes ago
Changing state of subset of light bulbs
Miquel-point   0
an hour ago
Source: KoMaL A. 907
$2025$ light bulbs are operated by some switches. Each switch works on a subset of the light bulbs. When we use a switch, all the light bulbs in the subset change their state: bulbs that were on turn off, and bulbs that were off turn on. We know that every light bulb is operated by at least one of the switches. Initially, all lamps were off. Find the biggest number $k$ for which we can surely turn on at least $k$ light bulbs.

Based on an OKTV problem
0 replies
Miquel-point
an hour ago
0 replies
Drawing excircle on circular paper with very strange ruler
Miquel-point   0
an hour ago
Source: KoMaL A. 906
Let $\mathcal{V}_c$ denote the infinite parallel ruler with the parallel edges being at distance $c$ from each other. The following construction steps are allowed using ruler $\mathcal V_c$:
[list]
[*] the line through two given points;
[*] line $\ell'$ parallel to a given line $\ell $at distance $c$ (there are two such lines, both of which can be constructed using this step);
[*] for given points $A$ and $B$ with $|AB|\ge c$ two parallel lines at distance $c$ such that one of them passes through $A$, and the other one passes through $B$ (if $|AB|>c$, there exists two such pairs of parallel lines, and both can be constructed using this step).
[/list]
On the perimeter of a circular piece of paper three points are given that form a scalene triangle. Let $n$ be a given positive integer. Prove that based on the three points and $n$ there exists $C>0$ such that for any $0<c\le C$ it is possible to construct $n$ points using only $\mathcal V_c$ on one of the excircles of the triangle.
We are not allowed to draw anything outside our circular paper. We can construct on the boundary of the paper; it is allowed to take the intersection point of a line with the boundary of the paper.

Proposed by Áron Bán-Szabó
0 replies
Miquel-point
an hour ago
0 replies
hard inequality omg
tokitaohma   5
N an hour ago by math90
1. Given $a, b, c > 0$ and $abc=1$
Prove that: $ \sqrt{a^2+1} + \sqrt{b^2+1} + \sqrt{c^2+1} \leq \sqrt{2}(a+b+c) $

2. Given $a, b, c > 0$ and $a+b+c=1 $
Prove that: $ \dfrac{\sqrt{a^2+2ab}}{\sqrt{b^2+2c^2}} + \dfrac{\sqrt{b^2+2bc}}{\sqrt{c^2+2a^2}} + \dfrac{\sqrt{c^2+2ca}}{\sqrt{a^2+2b^2}} \geq \dfrac{1}{a^2+b^2+c^2} $
5 replies
1 viewing
tokitaohma
May 11, 2025
math90
an hour ago
Non-decelarating sequence is convergence-inducing
Miquel-point   0
an hour ago
Source: KoMaL A. 905
We say that a strictly increasing sequence of positive integers $n_1, n_2,\ldots$ is non-decelerating if $n_{k+1}-n_k\le n_{k+2}-n_{k+1}$ holds for all positive integers $k$. We say that a strictly increasing sequence $n_1, n_2, \ldots$ is convergence-inducing, if the following statement is true for all real sequences $a_1, a_2, \ldots$: if subsequence $a_{m+n_1}, a_{m+n_2}, \ldots$ is convergent and tends to $0$ for all positive integers $m$, then sequence $a_1, a_2, \ldots$ is also convergent and tends to $0$. Prove that a non-decelerating sequence $n_1, n_2,\ldots$ is convergence-inducing if and only if sequence $n_2-n_1$, $n_3-n_2$, $\ldots$ is bounded from above.

Proposed by András Imolay
0 replies
Miquel-point
an hour ago
0 replies
Changing the states of light bulbs
Lukaluce   1
N an hour ago by sarjinius
Source: 2025 Macedonian Balkan Math Olympiad TST Problem 1
A set of $n \ge 2$ light bulbs are arranged around a circle, and are consecutively numbered with
$1, 2, . . . , n$. Each bulb can be in one of two states: either it is on or off. In the initial configuration,
at least one bulb is turned on. On each one of $n$ days we change the current on/off configuration as
follows: for $1 \le k \le n$, on the $k$-th day we start from the $k$-th bulb and moving in clockwise direction
along the circle, we change the state of every traversed bulb until we switch on a bulb which was
previously off.
Prove that the final configuration, reached on the $n$-th day, coincides with the initial one.
1 reply
Lukaluce
Apr 14, 2025
sarjinius
an hour ago
Proving radical axis through orthocenter
azzam2912   0
an hour ago
In acute triangle $ABC$ let $D, E$ and $F$ denote the feet of the altitudes from $A, B$ and $C$, respectively. Let line $DE$ intersect circumcircle $ABC$ at points $G, H$. Similarly, let line $DF$ intersect circumcircle $ABC$ at points $I, J$. Prove that the radical axis of circles $EIJ$ and $FGH$ passes through the orthocenter of triangle $ABC$
0 replies
azzam2912
an hour ago
0 replies
Ez induction to start it off
alexanderhamilton124   22
N an hour ago by Adywastaken
Source: Inmo 2025 p1
Consider the sequence defined by \(a_1 = 2\), \(a_2 = 3\), and
\[
a_{2k+1} = 2 + 2a_k, \quad a_{2k+2} = 2 + a_k + a_{k+1},
\]for all integers \(k \geq 1\). Determine all positive integers \(n\) such that
\[
\frac{a_n}{n}
\]is an integer.

Proposed by Niranjan Balachandran, SS Krishnan, and Prithwijit De.
22 replies
alexanderhamilton124
Jan 19, 2025
Adywastaken
an hour ago
GOTEEM #1: Incircle Concurrency
tworigami   30
N Mar 30, 2025 by ErTeeEs06
Source: GOTEEM: Mock Geometry Olympiad
Let $ABC$ be a scalene triangle. The incircle of $\triangle ABC$ is tangent to sides $\overline{BC}$, $\overline{CA}$, $\overline{AB}$ at $D, E, F$, respectively. Let $G$ be a point on the incircle of $\triangle ABC$ such that $\angle AGD = 90^\circ$. If lines $DG$ and $EF$ intersect at $P$, prove that $AP$ is parallel to $BC$.

Proposed by bluek
30 replies
tworigami
Jan 2, 2020
ErTeeEs06
Mar 30, 2025
GOTEEM #1: Incircle Concurrency
G H J
G H BBookmark kLocked kLocked NReply
Source: GOTEEM: Mock Geometry Olympiad
The post below has been deleted. Click to close.
This post has been deleted. Click here to see post.
tworigami
844 posts
#1 • 4 Y
Y by jhu08, Adventure10, Mango247, ismayilzadei1387
Let $ABC$ be a scalene triangle. The incircle of $\triangle ABC$ is tangent to sides $\overline{BC}$, $\overline{CA}$, $\overline{AB}$ at $D, E, F$, respectively. Let $G$ be a point on the incircle of $\triangle ABC$ such that $\angle AGD = 90^\circ$. If lines $DG$ and $EF$ intersect at $P$, prove that $AP$ is parallel to $BC$.

Proposed by bluek
Z K Y
The post below has been deleted. Click to close.
This post has been deleted. Click here to see post.
franchester
1487 posts
#2 • 3 Y
Y by jhu08, Adventure10, Mango247
A bunch of point definitions:
  • $D_1$ is the point diametrically opposite $D$ wrt the incircle
  • $D_2$ is the second intersection of $AD$ with the incircle
  • $T$ is the intersection of $DD_1$ with $AP$
  • $X$ is the intersection of $EF$ and $DD_1$
  • $E_1$ is the intersection of $DE$ with $AP$ and define $F_1$ analogously
  • $M$ is the midpoint of $BC$
  • $Q$ is the extouch point on $BC$

Since $DD_1$ is the diameter of the incircle, it is clear from the angle condition that $A,G, D_1$ are collinear. I will now show that $(T, X; D_1,D)=-1$, which follows from the following claim:

Claim: $F, D_1, E_1$ are collinear, and similarly for $E, D_1, F_1$.
Proof: Apply Pascal's on hexagon $FD_1GDEE$. We find that $FD_1\cap DE$, $A$, and $P$ are collinear. However, the intersection of $DE$ with $AP$ is $E_1$, so we must have $FD_1\cap DE=E_1$, or $F, D_1, E_1$ collinear. A similar argument with Pascal's on hexagon $ED_1GDFF$ yields $E, D_1, F_1$ collinear.

Now, applying Ceva-Menalaus to $\triangle DE_1F_1$, we easily get $(T, X; D_1, D)=-1$. Projecting through $A$, we obtain $(T, X; D_1,D)=(AT\cap BC, AX\cap BC; AD_1\cap BC, D)=-1$. However, it is well known that $AX\cap BC$ is the midpoint of $BC$, and that $AD_1\cap BC$ is the $A$ extouch point. Furthermore, we know that the intouch point and the extouch point are symmetric about the midpoint, so $(AT\cap BC, M; Q, D)=-1$ implies that $AT\cap BC$ is the point of infinity wrt $BC$, or $BC$ is parallel to $AT$. Since $A, T, P$ are collinear by definition, $AP$ is parallel to $BC$.
Z K Y
The post below has been deleted. Click to close.
This post has been deleted. Click here to see post.
cmsgr8er
434 posts
#3 • 5 Y
Y by Lol_man000, Tekashi6ix9ine, YerMom, jhu08, Adventure10
Let $M$ denote the midpoint of $\overline{BC}$ and $\overline{AM}\cap\overline{EF}=X$. Let $Y$ denote the antipode of $D$ with respect to the incircle, $\omega$. Let $\overline{GX}\cap \omega = Z.$ Note that $\angle YGD=\angle AGD=90^{\circ},$ so $A, Y, G$ are collinear.

It is well known that $\overline{AM}, \overline{EF}, \overline{DY}$ concur at $X.$

Claim: Quadrilateral $DZFE$ is harmonic.

Proof. Note that it suffices to prove that $A,Z,D$ are collinear.

Note $X$ lies on $\overline{EF}$, the polar of $A$. By La Hire's $A$ lies on the polar of $X$. Furthermore, by Brocard's on quadrilateral $DGYZ$, $\overline{GY}\cap\overline{DZ}$ lies on the polar of $X$ as well. Moreover, since $A\in\overline{GY}$ and on the polar of $X$, then $\overline{GY}\cap\overline{DZ}=A.$ Hence, $A,Z,D$ must be collinear.$\blacksquare$

Projecting $\omega$ onto line $\overline{EF}$ and then onto $\overline{BC}$ gives,
$$(D,Z;F,E)\stackrel{G}{=} (P,X;F,E)\stackrel{A}{=}(\overline{AP}\cap\overline{BC}, M;B,C)=-1.$$For which it is well known then that $\overline{AP}\cap\overline{BC} = \infty,$ so $\overline{AP}\parallel \overline{BC}.$
This post has been edited 3 times. Last edited by cmsgr8er, Nov 16, 2020, 2:42 PM
Z K Y
The post below has been deleted. Click to close.
This post has been deleted. Click here to see post.
rocketscience
466 posts
#4 • 7 Y
Y by rzlng, Pluto1708, cmsgr8er, tree_3, jhu08, Adventure10, ehuseyinyigit
Let $I$ be the incenter, and let $D'$ be the antipode of $D$ on the incircle so that $A, D', G$ collinear. Let $X$ be the foot from $A$ to $DD'$, and let $P = AX \cap GD$ be the orthocenter of $\triangle AD'D$. Right angles imply $AXGD$ cyclic, so $PX \cdot PA = PG \cdot PD$. Then $P$ lies on the radical axis of the incircle and $(AIEFX)$, which is $EF$, meaning $P = EF \cap GD \cap AX$ and we are done.
Z K Y
The post below has been deleted. Click to close.
This post has been deleted. Click here to see post.
GeoMetrix
924 posts
#5 • 9 Y
Y by Aryan-23, amar_04, Kagebaka, tree_3, mijail, jhu08, MrOreoJuice, Adventure10, Mango247
Here is my solution for p1 i submitted.
[asy]
 /* Geogebra to Asymptote conversion, documentation at artofproblemsolving.com/Wiki go to User:Azjps/geogebra */
import graph; size(14cm); 
real labelscalefactor = 0.5; /* changes label-to-point distance */
pen dps = linewidth(0.7) + fontsize(10); defaultpen(dps); /* default pen style */ 
pen dotstyle = black; /* point style */ 
real xmin = 2.3613380392430985, xmax = 23.887380706934806, ymin = -9.061409326786983, ymax = 5.15416560376006;  /* image dimensions */
pen dtsfsf = rgb(0.8274509803921568,0.1843137254901961,0.1843137254901961); pen rvwvcq = rgb(0.08235294117647059,0.396078431372549,0.7529411764705882); 
 /* draw figures */
draw((7.06,2.8)--(3.46,-4.48), linewidth(1.2) + dtsfsf); 
draw((3.46,-4.48)--(16.02,-4.46), linewidth(1.2) + dtsfsf); 
draw((16.02,-4.46)--(7.06,2.8), linewidth(1.2) + dtsfsf); 
draw(circle((8.03017757107319,-1.6364997487962782), 2.8362193026272435), linewidth(1.2)); 
draw((8.034693838122354,-4.472715455671781)--(10.537308327920737,-2.9625598155596276), linewidth(1.2)); 
draw((7.06,2.8)--(20.121430537802492,2.820798456270373), linewidth(1.2) + dtsfsf); 
draw((7.06,2.8)--(11.445306161877651,-4.467284544328221), linewidth(1.2) + rvwvcq); 
draw((7.06,2.8)--(9.74,-4.47), linewidth(1.2) + rvwvcq); 
draw((10.537308327920737,-2.9625598155596276)--(20.121430537802492,2.820798456270373), linewidth(1.2)); 
draw((5.487822929371477,-0.37929140949323353)--(20.121430537802492,2.820798456270373), linewidth(1.2)); 
draw((8.025661304024027,1.199715958079226)--(8.034693838122354,-4.472715455671781), linewidth(1.2) + rvwvcq); 
 /* dots and labels */
dot((7.06,2.8),dotstyle); 
label("$A$", (7.113840965876333,2.9456495378540297), NE * labelscalefactor); 
dot((3.46,-4.48),dotstyle); 
label("$B$", (3.521507871332976,-4.336862299722183), NE * labelscalefactor); 
dot((16.02,-4.46),dotstyle); 
label("$C$", (16.073706777558403,-4.322884349937968), NE * labelscalefactor); 
dot((8.03017757107319,-1.6364997487962782),linewidth(4pt) + dotstyle); 
label("$I$", (8.09229745077141,-1.527294393094892), NE * labelscalefactor); 
dot((5.487822929371477,-0.37929140949323353),linewidth(4pt) + dotstyle); 
label("$F$", (5.548310590044209,-0.2692789125155078), NE * labelscalefactor); 
dot((8.034693838122354,-4.472715455671781),linewidth(4pt) + dotstyle); 
label("$D$", (8.09229745077141,-4.364818199290614), NE * labelscalefactor); 
dot((9.815712055935128,0.567135097534706),linewidth(4pt) + dotstyle); 
label("$E$", (9.867497073366765,0.6812216728111381), NE * labelscalefactor); 
dot((8.025661304024027,1.199715958079226),linewidth(4pt) + dotstyle); 
label("$Z$", (8.078319500987195,1.31022941310083), NE * labelscalefactor); 
dot((10.537308327920737,-2.9625598155596276),linewidth(4pt) + dotstyle); 
label("G", (10.594350462145966,-2.8551996225953533), NE * labelscalefactor); 
dot((20.121430537802492,2.820798456270373),linewidth(4pt) + dotstyle); 
label("$P$", (20.183224014117727,2.9316715880698143), NE * labelscalefactor); 
dot((11.445306161877651,-4.467284544328221),linewidth(4pt) + dotstyle); 
label("$K$", (11.502917198119967,-4.3508402495063985), NE * labelscalefactor); 
dot((8.027291358535031,0.17604172516803054),linewidth(4pt) + dotstyle); 
label("$L$", (8.078319500987195,0.2898390788531074), NE * labelscalefactor); 
dot((9.74,-4.47),linewidth(4pt) + dotstyle); 
label("$M$", (9.79760732444569,-4.364818199290614), NE * labelscalefactor); 
clip((xmin,ymin)--(xmin,ymax)--(xmax,ymax)--(xmax,ymin)--cycle); 
 /* end of picture */
[/asy]
Let $Z$ be the $D-$antipode w.r.t $\odot(I)$ . Clearly $G=AZ \cap \odot(I)$. Now let $ZD \cap EF=L$ . Its well known (also egmo lemma 4.17) that $AL \cap BC=M$ is the midpoint of $BC$. Now note that $-1=(Z,G;F,E)\overset{D}{=}(F,E;L,P)$. Now let $AP \cap BC =T$. Note that $-1=(F,E;L,P)\overset{A}{=}(B,C;M,T)$ but its well known that $(B,C,M;P_{\infty})$ is harmonic $\implies T=P_{\infty} \implies AP \parallel BC$ . Done $\blacksquare$
Z K Y
The post below has been deleted. Click to close.
This post has been deleted. Click here to see post.
math_pi_rate
1218 posts
#6 • 5 Y
Y by AlastorMoody, amar_04, Pluto04, jhu08, Adventure10
Sharygin 2018 CR P20
Z K Y
The post below has been deleted. Click to close.
This post has been deleted. Click here to see post.
Aryan-23
558 posts
#7 • 2 Y
Y by jhu08, Adventure10
It is well known that $AM$ , $EF$ and $DD'$ concur where $D'$ is the point diametrically opposite $D$ wrt the incircle and $M$ is the midpoint of $BC$ . Note that $A,G,D$ are collinear due to obvious reasons . Call that concurrency point $Z$ . Let the parallel from $A$ to $BC$ intersect $EF$ at $P'$.
Now we have
$$-1 = (BC;M\infty_{BC})\stackrel{A}{=}(FE;ZP')$$But since $(F,E;D',G) = -1$ (the quadrilateral is harmonic )
Hence we have
$$-1 = (F,E;D',G)\stackrel{D}{=}(FE;ZP)$$So $P'=P$ , done
Z K Y
The post below has been deleted. Click to close.
This post has been deleted. Click here to see post.
AwesomeYRY
579 posts
#8 • 3 Y
Y by jhu08, Adventure10, Mango247
Simple solution:

We let Q be the intersection of the line through A parallel to BC and the circle with diameter of AD.

We know that Q is on circle with diameter AD or (AGD), so AQD=AQI=90 degrees where I is the incenter. Thus, Q lies on the circle with diameter AI. This is the same circle as AEF as AEIF is obviously cyclic. Thus, AGEF is cyclic. Taking the radical axis theorem on the circle with (AQGD), (AQEIF), (FEGD) which gives us that AQ, EF, GD are concurrent, and thus we have achieved the result that we wanted.
Z K Y
The post below has been deleted. Click to close.
This post has been deleted. Click here to see post.
BestChoice123
1119 posts
#9 • 1 Y
Y by jhu08
AwesomeYRY wrote:
Simple solution:

We let Q be the intersection of the line through A parallel to BC and the circle with diameter of AD.

We know that Q is on circle with diameter AD or (AGD), so AQD=AQI=90 degrees where I is the incenter. Thus, Q lies on the circle with diameter AI. This is the same circle as AEF as AEIF is obviously cyclic. Thus, AGEF is cyclic. Taking the radical axis theorem on the circle with (AQGD), (AQEIF), (FEGD) which gives us that AQ, EF, GD are concurrent, and thus we have achieved the result that we wanted.

Why $\angle AQI=90^{\circ}$? Can someone explain pls? :huh:
This post has been edited 1 time. Last edited by BestChoice123, Mar 16, 2020, 1:34 AM
Z K Y
The post below has been deleted. Click to close.
This post has been deleted. Click here to see post.
jayme
9792 posts
#10 • 1 Y
Y by jhu08
Dear Mathlinkers,
a way of a proof can be seen on

https://artofproblemsolving.com/community/c6t48f6h1621417_sharygin_cr_p20

Sincerely
Jean-Louis
Z K Y
The post below has been deleted. Click to close.
This post has been deleted. Click here to see post.
pad
1671 posts
#11 • 4 Y
Y by mathlogician, anonman, jhu08, Mango247
Diagram
Let $Q=(AEFI) \cap (AD)$.

Claim: $AQ\parallel BC$.
Proof: We know $\angle AQD=90$ and $\angle AQI=90$ since $(AEFI)$ has diameter $AI$. Therefore, $Q,I,D$ are collinear. But $ID\perp BC$, so $QD\perp BC$. Finally, since $\angle AQD=90$, we conclude that $AQ\parallel BC$. $\square$

Now, radical axis on $(AEFIQ), (DEF), (AD)$ tells us that $AQ, EF, DG$ concur. Since $AQ$ is the line through $A$ parallel to $BC$, we are done.
Z K Y
The post below has been deleted. Click to close.
This post has been deleted. Click here to see post.
mathlogician
1051 posts
#12 • 2 Y
Y by jhu08, Mango247
wait can someone check if this works? seems really sketchy

Let $M$ be the midpoint of $BC$, let $AG$ intersect the incircle at point $X$, and let $Q = XG \cap EF$. Note that $EF, AM, DX$ concur, and call the concurrence point $T$. Note that $A,X,G$ are collinear, so by tangents from $A$ note that $EFXG$ is harmonic. Now let $P'$ be the point on $EF$ such that $AP' \parallel BC$. It is obvious that $(B,C;M, P_{\infty}) = (F,E;T,P) = -1$ by projecting through $A$. But also by projecting through point $D$, $(E,F;X,G) = (E,F;T,P) = -1$ thus $P = P'$ and we win.

wait yay seems identical to something else in this thread
This post has been edited 1 time. Last edited by mathlogician, Jun 19, 2020, 12:02 AM
Z K Y
The post below has been deleted. Click to close.
This post has been deleted. Click here to see post.
kevinmathz
4680 posts
#13 • 1 Y
Y by jhu08
Let $H$ be where $AG$ hits the incircle of $\triangle ABC$. Since $\angle AGD = \angle HGD = 90^{\circ}$, then $H$ is the other point where the perpendicular line to $BC$ passing through $D$ hits the incircle of $\triangle ABC$. Letting $M$ be the midpoint of line $BC$ and $H'$ be where $EF$ hits $DH$, we have $A, H', M$ are collinear. Since $AE$ and $AF$ are tangents and $A, G, H$ are collinear, then $(HG; EF) = -1$ so taking projectivity with respect to point $D$ and line $EF$ we get $(H'P; EF) = -1$ and now with respect to $A$ and line $BC$, letting $AP$ hit $BC$ at $Q$, we have $(MQ; CB) = -1$ so since $BM=CM$ then $Q = \infty$ meaning that $AP$ and $BC$ are parallel.
Z K Y
The post below has been deleted. Click to close.
This post has been deleted. Click here to see post.
srijonrick
168 posts
#14 • 3 Y
Y by A-Thought-Of-God, jhu08, Mango247
More storage :D
tworigami wrote:
Let $ABC$ be a scalene triangle. The incircle of $\triangle ABC$ is tangent to sides $\overline{BC}$, $\overline{CA}$, $\overline{AB}$ at $D, E, F$, respectively. Let $G$ be a point on the incircle of $\triangle ABC$ such that $\angle AGD = 90^\circ$. If lines $DG$ and $EF$ intersect at $P$, prove that $AP$ is parallel to $BC$.

Proposed by bluek

Solution: Let $D'$ be the antipode of $D$ w.r.t $\odot(I), \overline{AP} \cap \overline{BC} = X$ and $\overline{DD'} \cap \overline{EF} = K.$ Now, by Incenter Concurrency Lemma we have $\overline{AK}$ passing through the midpoint of $BC$, let's say $M.$

Since $\angle AGD = 90^{\circ}$, so $\overline{AG}$ passes through $D'$. Next, as $AE, AF$ are tangent to $\odot(I)$ and $A,D',G$ are collinear, hence, $D'GEF$ is harmonic due to Harmonic Quad Lemma. Thus, $$-1=(D',G;F,E)\stackrel{D}= (K,P;F,E)\stackrel{A}=(M,X;B,C).$$But by Midpoints and Parallel Lines Lemma $(B,C;M,P_{\infty})=-1,$ where $P_{\infty}$ is the point at infinity along $\overline{BC}.$ Hence, $X=P_{\infty} \implies AP \parallel BC. \quad\square$
Z K Y
The post below has been deleted. Click to close.
This post has been deleted. Click here to see post.
djmathman
7938 posts
#15 • 1 Y
Y by jhu08
Let $Q$ be the antipode of $D$ with respect to the incircle and set $R = DM\cap EF$; clearly, $A$, $Q$, and $G$ are collinear. Then $GEMF$ is a harmonic quadrilateral, so
\[
-1 = (G,M;E,F) \stackrel{D}=(P,R;E,F) \stackrel{A}=(PA\cap BC, AR\cap BC; B,C).
\]But it's known that $AR$ is actually the $A$-median of $\triangle ABC$, so this last equality is only possible when $PA\cap BC = \infty_{BC}$ -- that is, when $PA\parallel BC$.
Z K Y
The post below has been deleted. Click to close.
This post has been deleted. Click here to see post.
ike.chen
1162 posts
#16 • 1 Y
Y by jhu08
Let $l$ be the line through $A$ parallel to $BC$, $D'$ be the antipode of $D$ (with respect to the incircle), and $N = l \cap DD'$. The desired conclusion is equivalent to showing $l$, $EF$, and $DG$ are concurrent.

The angle condition implies that $AG$ passes through $D'$. It's also easy to see $l \perp DD'$ by parallel lines.

Claim: $ANEIF$ and $ANGD$ are cyclic.

Proof. Because $N, D', I, D$ are collinear, $$\angle AND = \angle ANI = 90^{\circ} =  \angle AFI = \angle AEI$$proving the first claim.

Observe $\angle AGD = \angle AND = 90^{\circ}$ which proves our second claim. $\square$

Now, it follows that $AN = l$, $EF$, and $DG$ are concurrent the Radical Center of $(ANEIF)$, $(ANGD)$, and $(DGEF)$. $\blacksquare$

Note: If the centers of the $3$ circles are collinear, then the $3$ lines concur at infinity.
Z K Y
The post below has been deleted. Click to close.
This post has been deleted. Click here to see post.
Mathscienceclass
1241 posts
#17 • 1 Y
Y by jhu08
All antipodes, poles, and polars are taken w.r.t. incircle. With $M$ as the midpoint of $BC$, first establish the following:

Lemma, $M$, $I$, midpoint of $AD$ are collinear.
Proof. Let $D'$ be the antipode of $D$ w.r.t. incircle and $K$ the $A$-extouch. $A$, $D'$, $K$ are collinear by a common lemma. Since $M$ is the midpoint of $D$ and $K$ (isotomic), we are done by a $\frac{1}{2}$ homothety at $D$. $\square$

Since the midpoint of $AD$ is the center of $(AD)$, we know that $M$ lies on the perpendicular bisector of $DG$. Thus $MG$ is also tangent, so $P$ lies on the polar of $M$. By La Hire's $M$ lies on the polar of $P$.

Also we know that $P$ lies on $EF$, the polar of $A$, so $A$ lies on the polar of $P$ and thus line $AM$ is really the polar. Then $AM$ contains $P'$, the harmonic conjugate of $P$ w.r.t $EF$. This means $(B, C; M, P_{\infty}) \stackrel{A}{=} (E, F; P', AP_{\infty} \cap EF)$, implying $AP \parallel BC$. $\blacksquare$
Z K Y
The post below has been deleted. Click to close.
This post has been deleted. Click here to see post.
brianzjk
1201 posts
#18 • 2 Y
Y by jhu08, pad
The key observation is that the polar of $P$ with respect to the incircle is in fact $AM$, where $M$ is the midpoint of $BC$.

First, note that $A$ must lie on the polar of $P$, since the polar of $A$ is $EF$, so by La Hire, we get what we want. It's well known that $MG$ is tangent to the incircle, so the polar of $M$ is $DG$, but as $P$ lies on $DG$, then $M$ must lie on the polar of $P$ by La Hire.

Now, if we let $N=EF\cap AM$, we have
\[-1=(F,E;N,P) \stackrel{A} = (B,C;M,AP\cap BC)\]which implies $AP$ is parallel to $BC$, as desired.
Z K Y
The post below has been deleted. Click to close.
This post has been deleted. Click here to see post.
MrOreoJuice
594 posts
#19
Y by
Harmonics = scam. Call the incircle $\omega$, let $I$ be the incenter, $D'$ be the anitpode of $D$ on the incircle, since $\measuredangle AGD = \measuredangle D'GD$ so $A-D'-G$, now let $L=D'D \cap (AFIE)$. Note that $\measuredangle ALD = \measuredangle ALI = 90^\circ = \measuredangle AGD$ so $L \in (AGD)$. Also because of $90^\circ$ we have $AL \parallel BC$, finish by radical axis on $\{(ADG) , (AFIE) , \omega\}$.
Click to reveal hidden text
This post has been edited 1 time. Last edited by MrOreoJuice, Oct 17, 2021, 12:03 PM
Z K Y
The post below has been deleted. Click to close.
This post has been deleted. Click here to see post.
minusonetwelth
225 posts
#21
Y by
Let $M$ be the midpoint of $BC$. Furthermore, let $D'$ be the antipode of $D$. Clearly, as $\angle AGD=90^\circ$, the intersection of $AG$ with the incircle must be $D'$. Now, it is well known that $A$, $D'$ and the $A$-excirlce touch point $X$ are collinear. Furthermore, we know that $BD=CX$, so $DM=MX$. As $\angle DGX=90^\circ$, we have $MG=MD$, so $MG$ is tangent to the incircle. Therefore, $P$ lies on the polar of $M$, so $M$ lies on the polar of $P$. Also, $P$ lies on the polar of $A$, so $A$ lies on the polar of $P$. It follow that $AM$ is the polar of $P$, so if $K\coloneq EF\cap AM$. By projecting from $A$ onto $BC$, $-1=(F,E;K,P)=(A,C;M,P'=BC\cap AP)$, so $P'$ is the point at infinity, and thus $AP\parallel BC$, as desired.
Z K Y
The post below has been deleted. Click to close.
This post has been deleted. Click here to see post.
HamstPan38825
8866 posts
#22
Y by
Let $H$ be the $D$-antipode and $K = \overline{EF} \cap \overline{DI}$. By incenter concurrency lemma $\overline{AK}$ bisects $\overline{BC}$. Hence $$-1 = (EF;GH) \stackrel D= (EF; PK) \stackrel A= (CB; \overline{AP} \cap \overline{BC}, \overline{AK} \cap \overline{BC}).$$The previous observation implies $\overline{AP} \cap \overline{BC} = \infty_{BC}$, as needed.
Z K Y
The post below has been deleted. Click to close.
This post has been deleted. Click here to see post.
hukilau17
288 posts
#23
Y by
Complex numbers can also put this one away very quickly. Take the incircle as the unit circle, so that
$$|d|=|e|=|f|=1$$$$a=\frac{2ef}{e+f}$$$$b=\frac{2df}{d+f}$$$$c=\frac{2de}{d+e}$$$$g=\frac{d+a}{d\overline{a}+1} = \frac{de+df+2ef}{2d+e+f}$$$$p = \frac{dg(e+f) - ef(d+g)}{dg-ef} = \frac{d^2e^2+d^2f^2-2e^2f^2}{(e+f)(d^2-ef)}$$Then
$$p-a = \frac{d^2(e-f)^2}{(e+f)(d^2-ef)}$$$$b-c = -\frac{2d^2(e-f)}{(d+e)(d+f)}$$so that
$$\frac{a-p}{b-c} = \frac{(d+e)(d+f)(e-f)}{(e+f)(d^2-ef)}$$which is equal to its conjugate and thus real. $\blacksquare$
Z K Y
The post below has been deleted. Click to close.
This post has been deleted. Click here to see post.
Om245
164 posts
#24 • 1 Y
Y by GeoKing
If $D'$ is antipode of $D$ in incircle and $M$ is midpoint of $BC$ then it well know that $A-D'-G$ and $MQ=MD$.

Consider pole of $DG$ and $EF$ with respect to incircle which are $A$ and $M$ respectively and hence $AM$ is polar of $P$.
By well know fact we know $EF,AM,ID$ are concurrent at one point $X$.

Polar of $X$ is parallel to $BC$ as $ID \perp BC$. From fact that $X$ lie on $AM$ we get $AP \parallel BC$.
Z K Y
The post below has been deleted. Click to close.
This post has been deleted. Click here to see post.
eibc
600 posts
#25
Y by
By radical center lemma on $(AD), (AEIF)$, and the incircle, we find that $PA$, $(AEIF)$, and $(AD)$ meet again at some point $X \neq A$. Since $\overline{AX} \perp \overline{XI}$ and $\overline{AX} \perp \overline{XD}$, we have $X$, $I$, and $D$ collinear. But because $\overline{XID} \perp \overline{BC}$ and $\overline{AP} \perp \overline{XID}$, we have $\overline{AP} \parallel \overline{BC}$, as desired.
This post has been edited 1 time. Last edited by eibc, Mar 10, 2024, 8:22 PM
Z K Y
The post below has been deleted. Click to close.
This post has been deleted. Click here to see post.
ismayilzadei1387
219 posts
#26
Y by
It is very intersting

first $DI \cap EF=J$
$DI \cap AG=K$
so $LFDE$ is harmonic if we take pencil from $A$ it quickly implies
$(F,E;J,P)=-1$
So we can take some intersection point which will be infinity point soon
from easy lemma $AJ \cap BC$ at midpoint of $BC$
$AP \cap BC = R$
$(B,C;M,R)=-1$
also we know that
$(B,C;M,Q_{\infty})=-1$
so if 3 points in parantheses are coincide
then the forth ones will be coincide
$R \equiv Q_{\infty}$
and we are done
This post has been edited 2 times. Last edited by ismayilzadei1387, Mar 12, 2024, 6:33 PM
Z K Y
The post below has been deleted. Click to close.
This post has been deleted. Click here to see post.
bjump
1029 posts
#27
Y by
XIOO 10 min solve let $\overline{AG}$ intersect the incircle of $\triangle ABC$ again at $H$ then since $90^\circ = \angle AGD = \angle AHD$. $H$ is the $D$-antipode on the incircle. Now pascal on $HHFDDE$ gives $IJ \parallel BC$, and pascal on $FFHEED$ gives $AIJ$ are collinear. Now brokard gives that $\overline{AIJ}$ is the pole of $K=\overline{EF} \cap \overline{HD}$. Now $-1=(FE; HG) \stackrel{D}= (FE; KP)$ so $P$ lies on the polar of $K$ and as $\overline{AIJP} \parallel \overline{BC}$ we are finished.

Heavily rushed cuz my brother is making me do stuff
This post has been edited 1 time. Last edited by bjump, Aug 2, 2024, 12:32 AM
Z K Y
The post below has been deleted. Click to close.
This post has been deleted. Click here to see post.
Eka01
204 posts
#28 • 1 Y
Y by Sammy27
We define:- $M$ as midpoint of $BC$ ,$I$ as the incenter and $DI \cap EF=Q$, .
All poles and polars with respect to $(DEF)$.
It is easy to see that $EF$ is polar of $A$, $AP$ is polar of $Q$ since the required polar is perpendicular to $IQ$ and passes through $A$ due to La Hire's [Since $P$ lies on polar of $A$, $A$ must lie on polar of $P$.]
and it is well known that $DG$ is polar of $M$ because $M$ is circumcenter of $\Delta DGX$ where $X$ is the $A$ extouch point and $MD$ is tangent to the incircle. So taking the polar duality, it suffices to show that $A, Q,M$ are collinear but this is well known and hence we are done.
This post has been edited 6 times. Last edited by Eka01, Aug 14, 2024, 10:10 AM
Z K Y
The post below has been deleted. Click to close.
This post has been deleted. Click here to see post.
dolphinday
1326 posts
#29
Y by
Let $M$ is the midpoint of $BC$.
Let $AM \cap EF = X$. It is well known that $X \in DI$.
Let the $D$ antipode on the incircle be $D'$ and notice that $\angle DGA = \angle DGD'$ so $G-D'-A$.
Since $(B, C; N, \infty_{BC}) \overset{A}= (F, E; X, A\infty_{BC} \cap EF) = -1$, it suffices to show that $(F, E; X, P) = -1$.
This is equivalent to $AN$ being the polar of $P$ wrt the incircle.
Notice that the polar of $A$ wrt the incircle is $EF$ so by La Hire's, $A$ lies on the polar of $P$. Then since $AD' \cap BC = T$ which is the extouch point, and $N$ is the circumcenter of $(DGT)$ we have $ND = NG$ which implies $NG$ is a tangent to the incircle. So this implies $P$ lies on the polar of $N$ so by La Hire's we have that the polar of $P$ is $AN$ as desired.
Z K Y
The post below has been deleted. Click to close.
This post has been deleted. Click here to see post.
shendrew7
796 posts
#30
Y by
Let $M$ be the midpoint of $BC$. With respect to the incircle, notice $A$ is the pole of $EF$ and $M$ is the pole of $DG$, so $P = EF \cap DG$ is the pole of $AM$. Thus
\[-1 = (E, F; EF \cap AM, P) \overset{A}{=} (B, C; M, AP \cap BC) \implies AP \parallel BC. \quad \blacksquare\]
Z K Y
The post below has been deleted. Click to close.
This post has been deleted. Click here to see post.
Aiden-1089
293 posts
#31
Y by
First note that $EF$ is the polar of $A$.
Let $I$ be the incentre, $D'$ be the antipode of $D$, and $U$ be the $A$-extouch point. Then $A,D',G,U$ collinear.
Let $M$ be the midpoint of $BC$. Then by a homothety at $D$ with scale factor $2$ we have that $D'G // IM$, so $DG // IM$. Since $MD$ is tangent to the incircle we have that $DG$ is the polar of $M$.
It follows that the polar of $P=EF \cap DG$ is $AM$, so $-1=(E,F;P,EF \cap AM) \stackrel{A}{=} (C,B;AP \cap BC,M)$, from which it follows that $AP//BC$.
Z K Y
The post below has been deleted. Click to close.
This post has been deleted. Click here to see post.
ErTeeEs06
64 posts
#32
Y by
Let $D'$ be reflection $D$ in $I$, then it is wellknown that $A, D, G$ are collinear. Let $X=DE\cap D'F$ and $Y=DF\cap D'E$ and $Z=DD'\cap EF$. By Pascal on $DGD'EFF$ and $DGD'FEE$ we see that line $AP$ is the same line as $XY$. Brocard gives $ZI\perp XY$, but since $ZI$ is the same line as $DI$ we have $ZI\perp BC$ and combining gives $BC\parallel XY$ or $BC\parallel AP$ and we are done.
Z K Y
N Quick Reply
G
H
=
a